LSAT and Law School Admissions Forum

Get expert LSAT preparation and law school admissions advice from PowerScore Test Preparation.

 kristinaroz93
  • Posts: 160
  • Joined: Jul 09, 2015
|
#19154
I would like to start off by apologizing for all my posts lately, as I am taking the LSAT quite soon, and a feel a bit worried.

My question is in regards to the passage set #3: questions 1-6 of lesson 8 (pg. 8-18) from the lessons 5-8 booklet of powerscore materials. It is a comparative reading and I am curious about problem number 6. I got this question right, but I want to know if my logic behind it was correct. I was struggling between A and E since both were listed as solutions to the problem of antbiotic resistant bacteria. However, is A the better answer since passage b ultimately says, "Because evolving bacterial strains are likely to exhibit a growing immunity to vaccination, the development of more universally effective vaccines is needed before this becomes an adequate solution for anitbiotics". And since there were no complaints regarding the effectiveness of bacteriophages, I went with answer A. And the explanation also says that answer A apparently better adresses the specific issue presented in lines 9-14, but I do not really see how. Would the vaccines, if effective, not address this same issue?

Thanks in advance! I appreciate all the help this forum has provided me thus far.
 Lucas Moreau
PowerScore Staff
  • PowerScore Staff
  • Posts: 216
  • Joined: Dec 13, 2012
|
#19163
Hey, kristinaroz,

Don't worry about posting here a lot - that's what we're here for! :-D We'll answer as many of your questions as we can.

The problem with vaccinations lies in the first paragraph of Passage B, as it talks about how bacteria are developing resistance to vaccinations. This implies that a different form of treatment is needed.

Your hunch with answer choice A is correct: rather than rely on vaccinations, which bacteria are becoming increasingly likely to be increasingly immune to, we should turn to bacteriophages, which bacteria are not resistant to and which do not cause resistance to form.

Hope that helps,
Lucas Moreau
 Administrator
PowerScore Staff
  • PowerScore Staff
  • Posts: 8919
  • Joined: Feb 02, 2011
|
#27234
Complete Question Explanation

The correct answer choice is (A)

To respond to this question, we should begin by looking at the referenced problem in Passage A: that of plasmid-exchange, which allows for rapid genetic transfer and quick cultivation of resistant bacteria populations, even from what started as a single bacterium. To respond to this problem, the author of Passage B would likely suggest the first solution presented in Passage B: that of bacteriophages, which invade a bacterium and cause it to self-destruct. This would solve Author A’s problem of the spread and cultivation of resistant bacteria. Correct answer choice (A) provides this response. Incorrect answer choices (B), (D), and (E) present prospective solutions to the general problem of bacterial resistance, but they all fail to respond to the specific concern referenced by this question. Incorrect answer choice (C) provides a new concept, not presented in either passage: a hybrid of the two separate approaches of vaccination and use of localized bioactive phytochemicals.

Get the most out of your LSAT Prep Plus subscription.

Analyze and track your performance with our Testing and Analytics Package.